This is a MBT question, since the question stem specifies: The statements above logically commit the politician to which one of the following conclusions?

Our stimulus begins with the conditional indicator unless. If we translate this by negating one of the ideas and making it the sufficient condition, we should get something equivalent to: If our nation does not redistribute wealth, then we cannot alleviate economic injustice AND eventually there will be intolerable economic inequities. This dovetails nicely into the next sentence’s conditional, which tells us that intolerable economic inequities is enough for those who suffer from injustice to resort to violence. Having now given us this set of conditionals about the dire consequences of not redistributing wealth, the politician concludes with a rule; it is our nation’s responsibility to do whatever is necessary to alleviate conditions that lead to violent attempts at social reform.

It should be clear how this rule relates to what we are already told. Based on the preceding two sentences we know that not redistributing wealth means there will be intolerable economic inequities which will in turn necessarily lead to violent attempts at social reform. The lack of redistribution is therefore a condition which necessarily gives rise to violent attempts at social reform, triggering the politicians rule, and consequently it must be true that it is our nation’s responsibility to redistribute wealth.

Answer Choice (A) We haven’t been told anything about whether violent attempts at reform can be justified, we only know that they inevitably arise under certain conditions.

Correct Answer Choice (B) This is exactly what we should have noticed in our pre-phrasing. Whenever the stimulus of a MBT question contains a conditional rule, we should be thinking about whether the sufficient condition is triggered anywhere else in the stimulus.

Answer Choice (C) The stimulus tells us nothing about political expediency and whether it must be chosen over abstract moral principles.

Answer Choice (D) This is consistent with our politician’s statements, but only because they deal with specifically a case that will lead to intolerable social conditions, and mention nothing about what to do if they don’t. We can’t conclude from what the politician says that economic injustice isn’t a problem in other cases.

Answer Choice (E) This is a classic case of confusing necessity for sufficiency. Just because redistribution is required for economic justice does not mean it is enough for economic justice.


Comment on this

We’ve got a MBT question here, as the question stem asks: If the statements in the argument are all true, which one of the following must also be true on the basis of them?

This stimulus is all about constitutions and what can make them liberal. It begins by denying the view that whether a constitution is written or unwritten inherently makes them more or less liberal. The big takeaway here should be that just knowing a constitution is written isn’t enough to know whether or not it is liberal. The next sentence further emphasizes this point with the conditional indicator until; a written constitution by itself is just a piece of paper, it only becomes meaningful when its content is (i) interpreted and (ii) applied. Put otherwise, if a written constitution hasn’t been interpreted and applied, then it isn’t a real constitution yet. These two sentences arguing against the importance of whether a constitution is written are followed up by a helpful definition of what a constitution is. A constitution isn’t the written or unwritten text, it is a sum of procedures which together legitimize and control the state’s use of power. Our final sentence begins with the conclusion indicator therefore, and from all this talk about what a constitution is and whether its being written matters, concludes that a written constitution is only liberal if it is (i) interpreted and (ii) applied in a liberal way.

This is a four star question, and it is easy to see why; unlike a lot of MBT true questions the underlying logic of the stimulus isn’t made super clear. The key to getting this question correct is recognizing the essential claims the stimulus makes about what makes something a constitution and what makes something a specifically liberal constitution. The key triggers are whether a constitution has been interpreted and applied, and whether this was in a liberal way. Let’s look at the answer choices and eliminate the ones that could be false:

Answer Choice (A) Since we’ve been told a written constitution is just a paper with words until it is put into action, it must be false that we could judge whether a written constitution is actually liberal just by reading it.

Correct Answer Choice (B) B is correct for the same reason that A is incorrect. It says the opposite of A, and tells us that a written constitution can’t be judged to be liberal by its written text alone, which is exactly what must be true given the definition of a constitution and the requirements to be liberal that the stimulus gives us. If a written constitution is just words on paper prior to being interpreted and applied, then we can’t know whether it is a liberal constitution without it being interpreted and applied.

Answer Choice (C) This answer makes a really broad claim that we have zero support for in the stimulus. All we’ve been told is that written constitutions are not inherently more liberal than unwritten ones because a constitution becomes real only through interpretation and application. We know nothing about whether or not there are advantages between the two kinds.

Answer Choice (D) We were only told that there is nothing inherently liberal about written constitutions, we cannot infer from that that there is something inherently liberal about unwritten constitutions.

Answer Choice (E) This answer choice is the most selected incorrect answer for this question. We are given the conditional statement that if a written constitution is liberal, then it is required that it be interpreted and applied in a liberal way. What this answer does which makes it incorrect is confuse the sufficient and necessary conditions of this statement, so that merely being interpreted in a liberal way is enough for a constitution to be liberal. If you chose this answer, it might be helpful to review conditional logic and specifically why we cannot conclude that jsut because the necessary condition is true that the sufficient condition is as well.


Comment on this

Here we have a MBT question, which we should recognize from the question stem: If the statements above are true, which one of the following must on the basis of them be true*?*

This question appears to be a lot more difficult than it really is; its difficulty lies in how dense with terminology the stimulus is. If we keep our cool, and focus on discerning the logical relations behind all this scientific jargon, we should have an easy time once we actually get to the answer choices. So don’t get lost in the noise, and make sure to anchor yourself in the conditionals and conjunctions as they show up.

Our first sentence tells us that these things called pyrrole molecules can combine to form polypyrroles. I don’t know about you, but I have never in my life heard of pyrrole molecules. Don’t let yourself be thrown off by this foreign language, what you should take away from this first sentence is that there are these things, pyrroles, and they can combine together. The next sentence continues with the weird language, bringing in this new thing, zeolites. As soon as we notice that the sentence begins with an if, we should look for a conditional relationship. The sentence gives us some information about polypyrrole combination when it is exposed to zeolite, and tells us there are two possible outcomes, conjoined by an OR that should jump at us. If we diagram this conditional, we should end up with something along the lines of: PP combination + Z → chains in Z or lumps on Z. So when we have these polypyrroles form near zeolite, they do so either in lumps on the zeolite, or chains within it. Interesting! The next sentence begins with the conditional indicator when, and we should once again use the logic to guide us through the language. It’s important to note in this conditional that the polypyrrole formation is the necessary condition, what triggers the rule. It’s not the case that polypyrrole formation is enough for the zeolite to change color, it is in fact required that if zeolite changes color from yellow to black, pyrrole formed either lumps on the zeolite or chains in it.

After all this explanation of polypyrrole formation and how it works around zeolites, we are given a phenomena; some yellow zeolite was submerged in pyrrole and turned black, without any pyrrole lumps forming. If we are pre-phrasing, we should recognize that we’ve been told pyrrole formation is required for zeolite to change from yellow to black, so it must have happened either in chains in the zeolite or in lumps on its surface. Since we are told there were no lumps, it must be true that the pyrrole formed chains in the zeolite. With this in mind, let’s look at the answer choices. As always on a MBT question, we should eliminate answers by considering whether it is possible for them to not be true given the information in the stimulus.

Answer Choice (A) This answer choice must be false, as we are explicitly told the zeolite was free of pyrroles before being submerged. If you selected this, you should work on reading the stimulus more carefully.

Answer Choice (B) B also must be false as we are told that no lumps formed on the zeolites surface after it had been submerged. Once again, you should be able to eliminate this answer quickly if you carefully read the stimulus.

Correct Answer Choice (C) This answer says exactly what we inferred in our pre-phrasing. Given the information in the stimulus, it is the only possible explanation for how the zeolite changed from yellow to black. Therefore, it must be true.

Answer Choice (D) Another answer choice that must be false. The third conditional the stimulus gives us requires that polypyrroles form if a zeolite changes from yellow to black. Since the zeolite was free of pyrrole before it was submerged, and changed from yellow to black after being submerged, there must have been some polypyrrole formation when it was submerged.

Answer Choice (E) This is the only incorrect answer choice that is not necessarily false. However, it is still not the case that it must be true. We are given no support to infer with certainty what quantity of the pyrrole entered the zeolites inner chamber.


Comment on this

This is a must be true question, as the question stem demands: If the statements above are all true, which one of the following must also be true*?*

The first sentence tells us that Angela will earn her Psychology degree once she completes two courses; one in experimental design, the other in developmental psychology. Unfortunately for Angela, experimental design is a pre-requisite of developmental psychology, and won’t be available till the following term, so Angela won’t finish her degree for at least two more terms. The first sentence tells us that the two courses are enough (i.e. sufficient) for Angela to get her degree. We should infer from the second sentence that they are also required (i.e. necessary) for Angela to get her degree, and that ED is required for DP. It’s important for us to recognize that Angela getting a psychology degree and Angela completing developmental psychology are both sufficient and necessary for each other, and that what we have is therefore a biconditional relationship between the two events. If we were to translate these requirements into a conditional chain, it would be something like: P Degree ←→ DP → ED. Since this is all the information the stimulus has for us, we should expect the correct answer to be something guaranteed by these combined conditionals. Let’s see what the answer choices have in store for us:

Answer Choice (A) We’ve been told nothing about how long the course will take.

Answer Choice (B) We’ve been told nothing about the relative difficulty of the two courses.

Answer Choice (C) We’ve been told nothing about the pre-requisites for experimental design, just that it is required for Angela’s degree and to take developmental psychology.

Answer Choice (D) We’ve been told about the requirements for Angela to get a degree, but we can’t infer that is true of anyone seeking a psychology degree in her university. Maybe there are several options to receive the degree, but Angela’s course selection has committed her to needing these two credits specifically.

Correct Answer Choice (E) Angela’s degree needs both DP and ED, but since DP already requires that she have completed ED, then she must have satisfied all the requirements for her degree if she completes DP.


Comment on this

We should recognize this as a must be true question, as the question stem states: If the statements above are true, which one of the following must also be true*?*

This stimulus is quite short for a MBT question. The first sentence tells us about the plan of an airport designed for private aircraft to cover its expenses by charging fees to private aircraft users. Unfortunately, the plan failed and the airport was unable to meet its expenses because the revenue from the fees was lower than expected. And that’s all we get! Since this is a MBT question, we know the correct answer is going to be a 100% guaranteed inference from just this information. The only thing we’ve really been told about is that the expenses were greater than both total revenue (superset) and specifically revenue from user fees (subset). The correct answer will have to somehow relate to this information. Let’s see what we get:

Answer Choice (A) We’ve been told nothing about where the county’s citizens live!

Answer Choice (B) The fact that there was any revenue from user fees strongly suggests this is at least partly false, and it certainly isn’t something we know for sure.

Answer Choice (C) This is just a total non sequitur, nothing was said about the airport’s construction.

Correct Answer Choice (D) If the airport was unable to pay its operating expenses, then its expenses must have been larger than its total revenue, and therefore it must be true that its expenses were greater than a subset of its total revenue.

Answer Choice (E) This requires a lot of assumptions as we know very little about the airport’s users.


Comment on this

This is a must be true question, since the stem asks: If the claims above are true, which one of the following must, on the basis of them, be true?

The first sentence begins by informing us that planetary bodies vary in what they’re composed of, but most of those in the Solar System have solid surfaces. It may be helpful to think of this sentence in terms of a superset, planetary bodies, and two overlapping subsets, planetary bodies in the solar system and planetary bodies with solid surfaces. The next sentence begins with the conditional indicator unless, and tells us that the renewal of the surface of a planetary body with a solid surface requires enough heat for volcanic activity. This is followed up with another conditional sentence, beginning with the indicator any, that tells that a solid surface planetary body that does not renew its surface will eventually become covered in craters like the moon. The stimulus ends by telling us some of the specifically old planetary bodies in the Solar System, such as Europa, have solid icy surfaces with very few meteorite craters. Interesting!

When we facing a MBT question with lots of conditionals and modifiers such as this one, its important to get a sense of the logic behind the text. We should look for how our conditionals can connect and relate to each other. In this case what we should notice is that if a planetary body doesn’t have a hot enough core, then it must be heavily marked by craters. Since we know some planetary bodies with solid surfaces are icy and not heavily marked by craters, then it must be true that some planetary bodies with icy surfaces have hot enough cores for volcanic activity. Let’s see if any of the answer choices resemble this pre-phrase:

Answer Choice (A) This is a good example of an answer choice where it is important to remember not to bring in outside knowledge to the LSAT. I’m sure many of you know that the moon isn’t icy, but it is still false that this must be true just based off our stimulus.

Answer Choice (B) Remember our different supersets and subsets. We’ve only been told that the renewal requirement applies to specifically solid surface planetary bodies, but this answer choice applies it to planetary bodies as a whole. It’s important to have picked up on the “such a planetary body” referential phrasing in the second sentence to avoid falling for this answer.

Answer Choice (C) We’ve been told nothing about the proportion of solid surface planetary bodies without lots of craters that are icy. For all we know they are all icy.

Answer Choice (D) We are only told about one of Jupiter’s moons, and that it is not heavily pockmarked.

Correct Answer Choice (E) This is exactly what we pre-phrased. If the only way for a solid surface planetary body to not be heavily marked by craters is to be hot enough for volcanic activity, and we’ve been told about a very cold solid surface planetary body that is not heavily marked by crates, then there must be at least some (i.e Europa) very cold planetary bodies with hot enough cores for volcanic activity.

This page shows a recording of a live class. We're working hard to create our standard, concise explanation videos for the questions in this PrepTest. Thank you for your patience!

Comment on this

This page shows a recording of a live class. We're working hard to create our standard, concise explanation videos for the questions in this PrepTest. Thank you for your patience!

This is a straight forward question stem for an NA question. The argument depends on the assumption in the answer. This is asking for necessity.

First off in this stimulus, that is a powerful sounding book. I don’t know of anything this convincing, so we’ve set a very extreme characteristic to this book. But remember that on the LSAT, we accept the assertions as true. So this book is all powerful in its persuasiveness. Members of the Earth Association gave away 2,000 copies last month. Given its persuasive power, that seems like a worthwhile thing to do for an environmental organization. “Thus” is introducing the conclusion here in the final line: The EA converted 2,000 people to the cause.

Well, a lot of problems might come to mind. First, it’s not enough for someone to own the book to be persuaded by it. It still needs to be read. Did any of these people they gave the books to read it? Also, to be converted, the recipients must not have already been environmentalists. Who are these people? The Earth Association better not have been distributing these at a convention for environmentalists, or I’m skeptical that they haven’t just been handing these out to people who were already environmentalists. Maybe these issues are obvious to some of us, maybe not to others. I do think these are particularly conspicuous compared to the average NA question, but we need not see these as problems. Whether you saw these or not, you still want to keep an open mind with the answer choices. There may very well be something else. With NA, there is almost always other directions a correct answer could take. For example, another NA here would be something like, “Copies of To Save the Earth are not printed in a font too small for any of the 2,000 recipients to read.” Bet no one predicted that, but it would be the right answer if provided. It goes to the same idea that each recipient actually read it, but its presented in a surprising way that may be difficult to recognize if we’re committed to looking for any answer in particular.

Answer Choice (A) Well that doesn’t have to be true. The more the merrier. We might have an issue if other organizations gave it to the same recipients, but this doesn’t say that. If it did, then these people would get this book with or without the Earth Association and so that could be a problem for their claim. If you selected this, did you think that’s what it said? Read carefully!

Answer Choice (B) This is wrong, but it’s a little tricky. Their “willingness” to buy it does not particularly matter. They could both be willing to buy it and not have bought it. I’d be willing to buy lots of things I have not actually bought. So just because they’d’ve been willing to buy it doesn’t mean they’d have obtained (and read) a copy. Furthermore, if they were willing to buy it, it doesn’t at all matter that they would have been willing to have bought it from the Earth Association. Any bookstore or online retailer or yard sale or anything else would be fine. This just doesn’t have to be true.

Answer Choice (C) Recycled paper? No. We might expect this book to be sustainably printed, but this has nothing to do with its persuasive power or whether or not the Earth Association has changed hearts and minds.

Correct Answer Choice (D) Here it is. If someone was already committed to the cause when the Earth Association gave them the book, then the Earth Association cannot claim to have converted that person to the cause they were already committed to.

Answer Choice (E) This is another slippery one. We do need each recipient to convert to the environmentalist cause, but that need not mean they embrace the specific brand of environmentalism advocated for by the Earth Association. That is an additional assumption which we are not at liberty to make and which prevents this answer from being necessary.


Comment on this